LSAT and Law School Admissions Forum

Get expert LSAT preparation and law school admissions advice from PowerScore Test Preparation.

 Administrator
PowerScore Staff
  • PowerScore Staff
  • Posts: 8919
  • Joined: Feb 02, 2011
|
#23562
Complete Question Explanation

Assumption. The correct answer choice is (B)

The problem with the stimulus argument is that it attacks the proposed law on the basis of who has supported it most vocally, generally bypassing discussion of the actual merits of the proposed law. While there is a brief mention that there are problems associated with the adoption of the law, we have no idea what those problems are, or whether they are outweighed by the benefits. Therefore, to support the stimulus argument, we must use the Defender Model, to defend against the outside argument that the benefits outweigh the problems.

Answer Choice (A): While the stimulus does mention that the supporters of the law are some of the wealthiest summer residents, income level is not really a relevant issue to the discussion. The real issue in the stimulus argument is that the summer residents will not have to deal with the problems resulting from the law; it has nothing to do with income level.

Answer Choice (B): This is the correct answer choice. This answer choice defends against the outside argument that, regardless of the summer residents not having to deal with any problems, the overall benefits of this law outweigh the problems. This answer choice dismisses the use of that point and is essential to the stimulus argument.

Answer Choice (C): While this answer choice would support the argument's underlying assertion that summer residents have much more to gain from the proposed law, it is not essential to the argument.

Answer Choice (D): Once again, this answer choice would support the underlying assertion of the stimulus argument, but it is not essential to the argument.

Answer Choice (E): The permanent residents may be opposing passage of the law for some reason completely removed from the fact that it better serves the interests of the summer residents. They could have been misled by an ad campaign, they could have been paid off, etc. We simply do not know their motivation, and therefore this answer choice is not essential to the argument.
 netherlands
  • Posts: 136
  • Joined: Apr 17, 2013
|
#9619
Hi there PS,

I'm not sure I understand why the answer to this question is not E. The issue is that the most vocal proponents are summer residents who wont have to deal with the problems a law will bring and will only receive the benefits. It concludes that anyone who supports it is solely interested in serving the summer residents' interests.

Well- don't we have to assume that none of the permanent residents also support it? Just because "the most vocal supporters" are the summer residents doesn't necessarily mean that all or that even the majority of the supporters are summer residents. And a lot of permanent residents do support it wouldn't that counter the argument that supporters of the law only support summer residents?

I feel like I could see how B strengthens the claim - but I don't see how its a necessary assumption or how it trumps E.
 Steve Stein
PowerScore Staff
  • PowerScore Staff
  • Posts: 1153
  • Joined: Apr 11, 2011
|
#9621
Hey Netherlands,

Thanks for your message--in that one, the author's conclusion is that supporters of the law are serving only the interests of a few outsiders, and causing problems for the permanent residents.

As you mentioned, we are dealing with an Assumption question, so the correct answer choice must present an assumption that is required by the author's argument.

In order to test the validity of any of the contender answer choices, we can apply the Assumption Negation technique; the correct answer choice, when taken away (or, logically negated), will weaken the author's argument.

The problem with answer choice E is that it is not an assumption that is necessary to the author's argument. What if the majority of year-rounders don't oppose the law? Maybe they don't even know about it, but this would not hurt the author's conclusion, which is specifically about the proposed law's supporters.

Answer choice B provides that the problems associated with this law outweigh any benefits to the island's permanent residents. The logically negated version of this choice, then, would provide that the problems do not outweigh the benefits, meaning that the law would provide benefits for the year-round residents that are at least as great, if not greater, than the problems it would cause.

This negated version would crush the author's argument that supporters of the bill were only serving the interests of a few outsiders.


I hope that's helpful! Please let me know whether this is clear--thanks!

~Steve
 cameron_bodell
  • Posts: 3
  • Joined: Jul 03, 2016
|
#28889
Hi there,

On this question, I understand how (B) could be the correct assumption, but it seems that if negated, the answer choice doesn't kill the argument. For example, even if the problems associated with the law DON'T outweigh an benefits, wouldn't there still be at least some problems for the permanent residents? And wouldn't it still be possible that anyone who supports the law is "serving only the interests of a few outsiders?" At face value, it just doesn't seem necessary that the author assume that the problems would outweigh the benefits, because it seems like the author's conclusion could remain intact even if answer choice (B) was negated. Let me know where I am going wrong here. Thanks!
User avatar
 Jonathan Evans
PowerScore Staff
  • PowerScore Staff
  • Posts: 726
  • Joined: Jun 09, 2016
|
#29107
Hi, Cameron,

Good question, as both Steve and the official explanation above describe, answer choice B is in fact an assumption necessary for the author's conclusion to be valid.

Focus on the conclusion here; it is very strong. Symbolically:

support proposed law :arrow: only benefits outsiders

Negate answer choice B.

If the benefits to the permanent residents actually outweighed the problems they would face, then the permanent residents would stand to benefit from the current law, the necessary condition "only benefits outsiders" would be negated, and the conclusion would be invalid.

I hope this concise explanation might have shed some light on this problem for you.

Get the most out of your LSAT Prep Plus subscription.

Analyze and track your performance with our Testing and Analytics Package.